Đến nội dung

sinh vien nội dung

Có 261 mục bởi sinh vien (Tìm giới hạn từ 07-05-2020)



Sắp theo                Sắp xếp  

#559290 Một số bài toán tính giới hạn dãy số sử dụng tổng Riemann

Đã gửi bởi sinh vien on 14-05-2015 - 10:22 trong Giải tích

Bài toán. (Putnam 1970 )Tính giới hạn

               $lim_{n\rightarrow \infty }\frac{1}{n^{4}}\prod_{i=1}^{2n}(n^{2}+i^{2})^{\frac{1}{n}}$




#559288 Đa thức + Số học

Đã gửi bởi sinh vien on 14-05-2015 - 09:34 trong Đa thức

Bài toán (Putnam 2008 ) Cho p là một số nguyên tố . Gỉa sử h(x) là một đa thức hệ số nguyên sao cho $\left \{ h(0),h(1),...,h(p^{2}-1) \right \}$ là một hệ thặng dư đầy đủ modulo $p^{2}$.

 Chứng minh rằng $\left \{ h(0),h(1),...,h(p^{3}-1) \right \}$ lập thành một hệ thặng dư đầy đủ modulo $p^{3}$




#559287 Một ví dụ nhỏ về sử dụng hệ thặng dư

Đã gửi bởi sinh vien on 14-05-2015 - 09:29 trong Số học

Bài toán ( Putnam 2007 ) Cho k là một số nguyên dương . Chứng minh rằng tồn tại các đa thức $P_{0}(n),P_{1}(n),...,P_{k-1}(n)$ ( phụ thuộc k ) sao cho với mọi số nghuyên n

              $\left \lfloor \frac{n}{k} \right \rfloor^{k}=P_{0}(n)+P_{1}(n)\left \lfloor \frac{n}{k} \right \rfloor+...+P_{k-1}(n)\left \lfloor \frac{n}{k} \right \rfloor^{k-1}$, trong đó $\left \lfloor x \right \rfloor$ dùng để chỉ số nguyên lớn nhất không vượt quá x.

Lời giải. Mấu chốt của chứng minh nằm ở nhận xét sau :

 Với mọi số nguyên n , Tồn tại  một số $j\in \left \{ 0,1,...,k-1 \right \}$ sao cho $\left \lfloor \frac{n}{k} \right \rfloor=\frac{n-j}{k}$.

  Thật vậy , theo thuật toán chia với mọi số nguyên n , ta đều có thể viết $n=qk+j$ , trong đó $j\in \left \{ 0,...,k-1 \right \}$ nên$q\leq \frac{n}{k}=q+\frac{j}{k}< q+1\Rightarrow \left \lfloor \frac{n}{k} \right \rfloor=q=\frac{n-j}{k}$

 

 Từ nhận xét trên ta thấy 

                $\prod_{j=0}^{k-1}\left ( \left \lfloor \frac{n}{k} \right \rfloor -\frac{n-j}{k}\right )=0$

Đến đây khai triển tích trên ta có ngay điều phải chứng minh




#559232 Các bài toán về hàm hình học

Đã gửi bởi sinh vien on 13-05-2015 - 21:07 trong Hình học

Bài toán (Putnam 2009). Gọi $\mathbb{P}$ là tập hợp các điểm trên mặt phẳng , $f:\mathbb{P}\rightarrow \mathbb{R}$ là một hàm số nhận giá trị thực thỏa mãn $f(A)+f(B)+f(C)+f(D)=0$, với mọi hình vuông ABCD .

    Chứng minh rằng f(P)=0 với mọi P

Lời giải. Gọi P là một điểm bất kỳ thuộc $\mathbb{P}$. Gọi ABCD là hình vuông nhận P làm tâm . Gọi E,F,G,H lần lượt là trung điểm của các cạnh AB ,BC,CD,DA. 

Từ điều kiện đề bài ta có 

   0=f(A)+f(B)+f(C)+f(D)

   0=f(E)+f(F)+f(G)+f(H)

   0=f(A)+f(E)+f(P)+f(H)

   0=f(B)+f(F)+f(P)+f(E)

   0=f(C)+f(G)+f(P)+f(F)

   0=f(D)+f(H)+f(P)+f(G)

 Cộng bồn đẳng thức cuối kết hợp với hai đẳng thức đầu ta có ngay kết quả




#559215 Đa thức + Số học

Đã gửi bởi sinh vien on 13-05-2015 - 20:28 trong Đa thức

Bài toán . ( Putnam 2011 ) Cho p là một số nguyên tố  lẻ .

Chứng minh rằng có ít nhất $\frac{p+1}{2}$ giá trị $n\in \left \{ 0,1,2... p-1 \right \}$ sao cho $\sum_{k=0}^{p-1}k!n^{k}$ không chia hết cho p.

Lời giải. Đặt $f(x)=\sum_{k=0}^{p-1}k!x^{k}\in F_{p}[x]$ là tập hợp các đa thức bậc p có hệ số thuộc trường $F_{p}$. Ta phân hoạch $F_{p}=Z\bigcup Z^{'}$, 

 trong đó $Z=\left \{ a\in F_{p};f(a)=0 \right \};Z^{'}=\left \{ a\in F_{p} ;f(a)\neq 0\right \}$

 Ta thấy $f(0)=1\Rightarrow 0\in Z^{'}\Rightarrow \left | Z \right |\geqslant 1$.

 Đặt $\left | Z \right |=p-r,\left | Z^{'} \right |=r\geqslant 1$.

 Xét đa thức $s(x)=\prod_{a\in Z^{'}}(x-a)\Rightarrow degs(x)=r$

   Dựa vào các ta định nghĩa thì đa thức $s(x)f(x)$ triệt tiêu ( luôn bằng 0 ) tại mọi điểm của trường $F_{p}$,

Do đó $s(x)f(x)=(x^{p}-x)h(x)$, trong đó h(x) có bậc r-1 và thuộc $F_{p}[x]$ ( Lưu ý đến bậc của hai vế + Định lý Fermat nhỏ ). Trong những tính toán tiếp theo có sử dụng đến đạo hàm hình thức .

$x\frac{d}{dx}(xf(x))=x\frac{d}{dx}\sum_{k=0}^{p-1}k!x^{k+1}=x\sum_{k=0}^{p-2}(k+1)!x^{k}=\sum_{k=1}^{p-1}k!x^{k}=f(x)-1$

$\Rightarrow x^{2}f^{'}(x)+xf(x)=f(x)-1\Rightarrow x^{2}f^{'}(x)+(x-1)f(x)+1=0$

  Lấy đạo hàm hình thức $s(x)f(x)=(x^{p}-x)h(x)$ ta được

      $s^{'}(x)f(x)+s(x)f^{'}(x)=\left ( x^{p}-x \right )h^{'}(x)-h(x)$ ( Chú ý loại bỏ các số hạng triệt tiêu trên trường $F_{p}$ )

 Từ các kết luận trên ta thấy

 $x^{2}s^{'}(x)f(x)=(x^{p}-x)x^{2}h^{'}(x)-x^{2}h(x)-s(x)x^{2}f^{'}(x)$

                          $=(x^{p}-x)x^{2}h^{'}(x)-x^{2}h(x)+s(x)((x-1)f(x)+1)$

                         $=(x^{p}-x)x^{2}h^{'}(x)-x^{2}h(x)+(x-1)(x^{p}-x)h(x)+s(x)$

                         $=\left ( x^{p}-x \right )(x^{2}h^{'}(x)-(x-1)h(x))-x^{2}h(x)+s(x)$

Ta thấy vế trái triệt tiêu trên $Z\bigcup \left \{ 0 \right \}$,  $x^{p}-x$ triệt tiêu trên $F_{p}$ nên đa thức $-x^{2}h(x)+s(x)$ triệt tiêu trên $Z\bigcup\left \{ 0 \right \}$.

Ta thấy

 $r+1=deg(-x^{2}h(x)+s(x))\geqslant \left | Z \right |+1=p-r+1\Rightarrow r\geqslant \frac{p+1}{2}$




#559180 Một số bài toán cơ bản

Đã gửi bởi sinh vien on 13-05-2015 - 18:40 trong Tài liệu và chuyên đề Đại số đại cương

Bài toán. Trên tập hợp S được trang bị một phép toán . ( kí hiệu theo lối nhân ) vừa giao hoán vừa kết hợp. Gỉa sử với mọi x, y thuộc S , tồn tại z thuộc S sao cho xz=y ( z phụ thuộc vào x,y )

 Chứng minh rằng Với mọi a,b,c thuộc S nếu ac=bc thì a=b 

Bài toán ( Putnam 1971). Cho S  là một tập hợp được trang bị phép toán $\circ$ thỏa mãn hai điều kiện sau

                       $i) x\circ x=x$ với mọi x thuộc S

                       $ii) (x\circ y)\circ z=(y\circ z)\circ x$ với mọi x,y,z thuộc S

Chứng minh : $\circ$ kết hợp và giao hoán




#559160 Kĩ thuật tổng hợp trong các bài toán giải tích

Đã gửi bởi sinh vien on 13-05-2015 - 17:56 trong Giải tích

Bài toán ( Putnam 2013 ). Cho các số thực $a_{0},a_{1},...a_{n}$ và số thực x thỏa 0 <x<1 , sao cho

                $\frac{a_{0}}{1-x}+\frac{a_{1}}{1-x^{2}}+\frac{a_{2}}{1-x^{3}}+...+\frac{a_{n}}{1-x^{n+1}}=0$

Chứng minh rằng:Đa thức $a_{0}+a_{1}y+a_{2}y^{2}+...+a_{n}y^{n}=0$ có nghiệm trong khoảng (0,1)

Lời giải. Gỉa sử ngược lại đa thức $f(y)=a_{0}+a_{1}y+a_{2}y^{2}+...+a_{n}y^{n}$ không có nghiệm trong khoảng (0,1 ) thì do f là hàm liên tục nên f giữ nguyên dấu trên khoảng (0,1) ( có nghĩa là chỉ mang dấu dương hoặc chỉ âm ) . Không mất tính tổng quát ta có thể giả sử f(y)>0 với mọi 0<y<1

 ( nếu f(y)<0 thì ta có thể xét đa thức -f(y) thì các giả thiết và kết luận của bài toán không đổi )

 Do đó :$f(x^{k})>0,k\geq 1.$ 

     Lưu ý đến bất đẳng thức :$0< x^{k}< x< 1$.

Quan trọng hơn cả là $f(x^{0})=f(1)=lim_{y\rightarrow 1^{-}}f(y)\geqslant 0$

  Ta có biến đổi sau

$\sum_{0\leq i\leq n}\frac{a_{i}}{1-x^{i+1}}=\sum_{0\leq i\leqslant n}\sum_{k\geq 0}a_{i}x^{(i+1)k}=\sum_{k\geq 0}\sum_{0\leq i\leq n}a_{i}x^{(i+1)k}=\sum_{k\geq 0}x^{k}\sum_{0\leq i\leq n}a_{i}x^{ik}=\sum_{k\geq 0}x^{k}f(x^{k})> 0$

 Lưu ý rằng theo nhận xét ở trên thì trong đẳng thức cuối cùng số hạng thứ nhất không âm còn tất cả các số hạng còn lại đều dương.

Bất đẳng thức trên mâu thuẫn với điều kiện ở đầu bài . Do đó ta được điều phải chứng minh




#559142 Bộ tài liệu ôn thi olympic môn giải tích

Đã gửi bởi sinh vien on 13-05-2015 - 17:23 trong Tài liệu, chuyên đề Toán cao cấp

Để cho thuận tiện mình có chuyển các file này sang định dạng pdf cho dễ xem

File gửi kèm  VIETMATHS.COM_Bai_tap_on_luyen_thi_Olympic_Toan_Tran_Van_Su.PDF   480.56K   2200 Số lần tảiFile gửi kèm  VIETMATHS.COM_Olympic_Toan_Sinh_vien_Giai_Tich.PDF   807.51K   2074 Số lần tải




#559119 Bộ tài liệu ôn thi olympic môn giải tích

Đã gửi bởi sinh vien on 13-05-2015 - 15:19 trong Tài liệu, chuyên đề Toán cao cấp

Bài tập giải tích của Kaczor - Nowak ( 2 cuốn tiếng việt + 1 cuốn tiếng anh )

Tập 1File gửi kèm  problem_analysis1.pdf   2.35MB   4522 Số lần tải

Tập 2 File gửi kèm  BaiTapGiaiTich-Tap2- KaczkorNowak-DoanChi-dich.pdf   2.06MB   4736 Số lần tải

Tập 3 File gửi kèm  Kaczor W.J., Nowak N.T. Problems in mathematical analysis 3. Integration (AMS, 2003)(ISBN 0821832980)(600dpi)(T)(356s)_MCetp_.pdf   12.9MB   5379 Số lần tải

Đề thi một số năm trên diễn đàn có khá nhiều ở đây chỉ nên một số bài mình sở hữu

 1993 - 2005 (  có  Giải tích + Đại số )File gửi kèm  06-10-2014 22.46.37tuyen tap de thi OLP SV toan quoc.pdf   1.27MB   2140 Số lần tải

 2006-2012 ( chỉ Gỉai tích ) File gửi kèm  VNMATH.COM-De Thi Loi Giai Olympic Toan SV Giai Tich 2006 - 2012.rar   759.92K   1991 Số lần tải

2013 ( Giaỉ tích + Đại số , file này tải lâu à nha ) File gửi kèm  7056d1fa-d643-4196-b129-10563fcb9535_kyyeuolympictoan2013.pdf   15.18MB   918 Số lần tải

2014 ( Giaỉ tích + Đại số ) File gửi kèm  OSV2014_Ky_yeu.pdf   2.87MB   1729 Số lần tải

2015  (chỉ Giaỉ tích )File gửi kèm  gt2015.pdf   5.33MB   1315 Số lần tải

 Cơ bản ôn nhiêu chắc cũng chủ đậu rồi




#559057 Danh sách các bài toán tích phân - chuỗi - tích vô hạn của Gradshteyn Ryzhik

Đã gửi bởi sinh vien on 13-05-2015 - 09:10 trong Giải tích

Bài toán. Tính tích phân $I=\int_{0}^{\infty }e^{-sx}\frac{sinxdx}{x}$, trong đó $s> 0$

Lời giải. Xét hàm $f(x,y)=e^{-sxy}sinx$ ,ta có:

$\int_{0}^{\infty }\int_{1}^{\infty }\left | f(x,y) \right |dydx=\int_{0}^{\infty }\int_{1}^{\infty }e^{-sxy}\left | sinx \right |dydx=\frac{1}{s}\int_{0}^{\infty }e^{-sx}\frac{\left | sinx \right |}{x}dx< \frac{1}{s}\int_{0}^{\infty }e^{-sx}dx< \infty$.

   Ta thấy

$\int_{0}^{\infty }\int_{1}^{\infty }f(x,y)dydx=\frac{1}{s}\int_{0}^{\infty }e^{-sx}\frac{sinxdx}{x}$

Mặt khác 

    $\int_{0}^{\infty }\int_{1}^{\infty }f(x,y)dydx=\int_{1}^{\infty }\int_{0}^{\infty }f(x,y)dxdy=\int_{1}^{\infty }\left ( \int_{0}^{\infty }e^{-sxy}sinxdx \right )dy$

  Ta tính tích phân phụ sau  : $\int_{0}^{\infty }e^{- ax}sinxdx=lim_{A\rightarrow \infty }\int_{0}^{A}e^{-ax}sinxdx$

$\int_{0}^{A}e^{-ax}sinxdx=\int_{0}^{A}e^{-ax}d(-cosx)=-e^{-ax}cosx _{0}^{A}-a\int_{0}^{A}e^{-ax}cosxdx$

$=-e^{-aA}cosA+1 -a\int_{0}^{A}e^{-ax}cosxdx$

$\int_{0}^{A}e^{-ax}cosxdx=\int_{0}^{A}e^{-ax}d(sinx)=e^{-ax}sinx_{0}^{A}+a\int_{0}^{A}e^{-ax}sinxdx=e^{-aA}sinA+a\int_{0}^{A}e^{-ax}sinxdx$

$\Rightarrow \int_{0}^{A }e^{-ax}sinxdx=\frac{1}{a^{2}+1}-\frac{e^{-aA}(cosA+asinA)}{a^{2}+1}$

  Mặt   khác $\left | cosA+asinA \right |\leq \sqrt{1+a^{2}}\Rightarrow \left | e^{-aA}(cosA+asinA) \right |\leq e^{-aA}\sqrt{1+a^{2}}\rightarrow 0,A\rightarrow \infty$

   Do đó $\int_{0}^{\infty }e^{-ax}sinxdx=lim_{A\rightarrow \infty }\left ( \frac{1}{a^{2}+1}-\frac{e^{-aA}(cosA+asinA)}{a^{2}+1} \right )=\frac{1}{a^{2}+1}$

Từ kết quả phụ này ta thấy

$\int_{0}^{\infty }\int_{1 }^{\infty }f(x,y)dydx=\int_{1}^{\infty }\frac{dy}{(sy)^{2}+1}=lim_{A\rightarrow \infty }\int_{1}^{A}\frac{dy}{(sy)^{2}+1}=lim_{A\rightarrow \infty }\frac{1}{s}arctan(sy)_{1}^{A}$

$=\frac{1}{s}\left ( \frac{\pi }{2} -arctans\right )=\frac{1}{s}arctan(s^{-1})$

 Kết luận :

                  $\int_{0}^{\infty }e^{-sx}\frac{sinxdx}{x}=arctan(s^{-1})$

 




#558727 Một số bài toán cơ bản

Đã gửi bởi sinh vien on 11-05-2015 - 13:06 trong Tài liệu và chuyên đề Đại số đại cương

Bài toán.(Putnam 2010). Cho G là một cấu trúc nhóm với phép toán *. Gỉa sử

i) G là tập con của tập hợp $R^{3}$

II) Với a,b $\in G$ thì hoặc $a\times b=a*b$ hoặc$a\times b=0$ , hoặc cả hai ( trong đó $\times$ là tích có hướng của hai vector trong $R^{3}$ )

 Chứng minh rằng $a\times b=0$ với mọi a,b $\in G$

Bài toán. Cho $S=\left \{ 3,5,7,... \right \}$. Với $x\in S$ , gọi $\delta (x)$ là số nguyên  dương thỏa $2^{\delta (x)}< x< 2^{\delta (x)+1}$. Với a, b $\in G$, ta định nghĩa $a*b=2^{\delta (a)-1}(b-3)+a$.

 Chứng minh rằng với $a,b,c\in G$ thì $(a*b)*c=a*(b*c)$

 




#558710 Danh sách các bài toán tích phân - chuỗi - tích vô hạn của Gradshteyn Ryzhik

Đã gửi bởi sinh vien on 11-05-2015 - 08:03 trong Giải tích

Bài toán . Tính tích phân $I=\int_{0}^{\infty }\frac{cosaxdx}{x^{2}+1}$

Lời giải . Ta sẽ sử dụng đến lý thuyết tích phân hàm biến phức. 

Xét tích phân $\int_{\Gamma }\frac{e^{ia\zeta }d\zeta }{\zeta ^{2}+1}$

trong đó $\Gamma =[-R,R]\bigcup\left \{ z,\left | z \right |=R,Imz\geqslant 0,R> 1 \right \}$

Ta viết lại hàm dưới dấu tích phân như sau:

$\frac{e^{ia\zeta }}{\zeta ^{2}+1}=\frac{e^{ia\zeta }}{\zeta +i}\frac{1}{\zeta -i}=\frac{f(\zeta )}{\zeta -i}$

Hàm f(z) chỉnh hình trong và trên $\Gamma$ ,áp dụng định lý Cauchy , ta được

$f(i)=\frac{1}{2\pi i}\int_{\Gamma }\frac{f(\zeta )d\zeta }{\zeta -i}\Rightarrow \frac{e^{i^{2}a}}{i+i}=\frac{1}{2\pi i}\int_{\Gamma }\frac{e^{ia\zeta }d\zeta }{\zeta ^{2}+1}$

hay $\int_{\Gamma }\frac{e^{ia\zeta }d\zeta }{\zeta ^{2}+1}=\pi e^{-a}$

Ta thấy 

                    $\int_{\Gamma }\frac{e^{ia\zeta }d\zeta }{\zeta ^{2}+1}=\int_{-R}^{R}\frac{e^{ia\zeta }d\zeta }{\zeta ^{2}+1}+\int_{C}\frac{e^{ia\zeta }d\zeta }{\zeta ^{2}+1}$

trong đó $C=\left \{ z,\left | z \right | =R,Imz\geq 0,R> 1 \right \}$

  Ta xét tích phân $\int_{C}\frac{e^{ia\zeta }d\zeta }{\zeta ^{2}+1}$

Đặt $\zeta =Re^{i\theta },\theta \in [0,\pi ]$, ta có

                      $\int_{C}\frac{e^{ia\zeta }d\zeta }{\zeta ^{2}+1}=iR\int_{0}^{\pi }\frac{e^{i\theta }e^{iaR(cos\theta +isin\theta )}d\theta }{R^{2}e^{2i\theta }+1}$.

 Ta có các đánh giá sau

                   $\left | e^{i\theta }e^{iaR(cos\theta +isin\theta )} \right |=e^{-aRsin\theta }\leq 1,\theta \in [0,\pi ]$

              và       $\left | R^{2}e^{2i\theta }+1 \right |\geq R^{2}-1$

nên     $\left | \int_{C}\frac{e^{ia\zeta }d\zeta }{\zeta ^{2}+1} \right |\leq \frac{R}{R^{2}-1}\int_{0}^{\pi }d\theta =\frac{\pi R}{R^{2}-1}\rightarrow 0,R\rightarrow \infty$

    Do đó:

$\int_{-\infty }^{\infty }\frac{e^{iax}dx}{x^{2}+1}=lim_{R\rightarrow \infty }\int_{-R}^{R}\frac{e^{ia\zeta }d\zeta }{\zeta ^{2}+1}=\pi e^{-a}-lim_{R\rightarrow \infty }\int_{C}\frac{e^{ia\zeta }d\zeta }{\zeta ^{2}+1}=\pi e^{-a}$

$\int_{-\infty }^{\infty }\frac{cosaxdx}{x^{2}+1}=Re\left ( \int_{-\infty }^{\infty }\frac{e^{iax}dx}{x^{2}+1} \right )=\pi e^{-a}\Rightarrow \int_{0}^{\infty }\frac{cosaxdx}{x^{2}+1}=\frac{\pi }{2}e^{-a}$




#558415 Danh sách các bài toán tích phân - chuỗi - tích vô hạn của Gradshteyn Ryzhik

Đã gửi bởi sinh vien on 09-05-2015 - 08:25 trong Giải tích

Bài toán . Tính tích phân

I=$\int_{0}^{\infty }e^{-x^{2}}cosaxdx$, trong đó a là hằng số dương

Lời giải . Khai triển Taylor hàm cosax

$cosax=1-\frac{\left ( ax \right )^{2}}{2}+\frac{\left ( ax \right )^{4}}{4!}+...+\frac{(-ax)^{2n}}{(2n)!}+...$

  Đặt $I_{n}=\int_{0}^{\infty }e^{-x^{2}}x^{2n}dx$. 

Ta thấy

$I_{n-1}=\int_{0}^{\infty }e^{-x^{2}}x^{2n-2}dx=\frac{e^{-x^{2}}x^{2n-1}}{2n-1}_{0}^{\infty }+\frac{2}{2n-1}\int_{0}^{\infty }e^{-x^{2}}x^{2n}dx=\frac{2}{2n-1}I_{n-1}$

$\Rightarrow I_{n}=\frac{2n-1}{2}I_{n-1}=\frac{(2n-1)(2n-3)}{4}I_{n-3}=...=\frac{(2n)!}{4^{n}n!}I_{0}$

 mà $I_{0}=\int_{0}^{\infty }e^{-x^{2}}dx=\frac{\sqrt{\pi }}{2}$$\Rightarrow I_{n}=\frac{(2n)!}{4^{n}n!}\frac{\sqrt{\pi }}{2}$

 Do đó 

I=$\frac{\sqrt{\pi }}{2}\sum_{n=0}^{\infty }(-1)^{n}\frac{a^{n}}{(2n)!}\frac{(2n)!}{4^{n}n!}=\frac{\sqrt{\pi }}{2}\sum_{n=0}^{\infty }\frac{\left ( -\frac{a^{2}}{4} \right )^{n}}{n!}=\frac{\sqrt{\pi }}{2}e^{-\frac{a^{2}}{4}}$




#558374 Danh sách các bài toán tích phân - chuỗi - tích vô hạn của Gradshteyn Ryzhik

Đã gửi bởi sinh vien on 08-05-2015 - 21:16 trong Giải tích

Mình post file lên trước và sẽ chứng minh một số kết quả sau.

File gửi kèm  Gradshteyn, Ryzhik. Tables of integrals, series, and products (5ed., AP, 1996)(1762s).pdf   4.61MB   814 Số lần tải




#558204 $\int \int \int_{B}\frac{x^{4}+2y^{4}}{x^{4}+4y^{4}+...

Đã gửi bởi sinh vien on 07-05-2015 - 17:00 trong Giải tích

Bài toán. Tính tích phân 

                                               I=  $\int \int \int_{B}\frac{x^{4}+2y^{4}}{x^{4}+4y^{4}+z^{4}}dxdydz$

trong đó $B=\left \{ (x,y,z);x^{2}+y^{2}+z^{2}\leq 1 \right \}$

Bài toán. Tính tích phân

                   I= $\int \int_{D}\left | xy \right |dxdy$

trong đó$D=\left \{ (x,y);x\geq 0,\left ( \frac{x^{2}}{a^{2}}+\frac{y^{2}}{b^{2}} \right )^{2}\leq \frac{x^{2}}{a^{2}}-\frac{y^{2}}{b^{2}} \right \}$

Bài toán. Tính tích phân

          I= $\int_{0}^{1}\int_{0}^{1}\int_{0}^{1}\frac{dxdydz}{\left ( 1+x^{2}+y^{2}+z^{2} \right )^{2}}$

Bài toán.Tính tích phân

     I= $\int_{-\infty }^{\infty }\int_{-\infty }^{\infty }\frac{dxdy}{\left ( p+ax^{2} +2bxy+cy^{2}\right )^{2}}$

trong đó p >0,a>0 và ac-b$^{2}$>0.




#558201 ứng dụng số phức để giải hệ phương trình vi phân

Đã gửi bởi sinh vien on 07-05-2015 - 16:43 trong Giải tích

Bài toán.  Giải hệ phương trình vi phân 

                                    $x^{''}-y^{'}+x=0$

                                    $y^{''}+x^{'}+y=0$

trong đó x(t) , y(t) là các hàm nhận giá trị  thực.

Lời giải. Nhân phương trình thứ hai cho i và cộng với phương trình thứ nhất ta được

$\left ( x+iy \right )^{''}+i\left ( x+iy \right )^{'}+\left ( x+iy \right )=0$

Bằng cách đặt  $z=x+iy$ đẳng thức trên trở thành một phương trình vi phân cấp 2 theo biến hàm mới - z.

                                                   $z^{''}+iz^{'}+z=0$

 Phương trình đặc trưng $\lambda ^{2}+i\lambda +1=0\Rightarrow \lambda _{1,2}=\frac{-1\pm \sqrt{5}}{2}i$

  Nghiệm tổng quát của phương trình có dạng

 $z(t)=\left ( a+ib \right )exp\left ( \frac{-1+\sqrt{5}}{2}it \right )+\left ( c+id \right )exp\left ( \frac{-1-\sqrt{5}}{2}it \right )$

          Lưu ý rằng ta đang xét x(t), y(t) là các hàm nhận giá trị thực. Thực hiện khai triển và đối chiếu phần thực , phần ảo ta thu được dạng tồng quát của nghiệm phương trình ban đầu

  $x(t)=acos\frac{-1+\sqrt{5}}{2}t-bsin\frac{-1+\sqrt{5}}{2}t+ccos\frac{-1-\sqrt{5}}{2}t-dsin\frac{-1-\sqrt{5}}{2}t$

$y(t)=asin\frac{-1+\sqrt{5}}{2}t+bcos\frac{-1+\sqrt{5}}{2}t+csin\frac{-1-\sqrt{5}}{2}t+dcos\frac{-1-\sqrt{5}}{2}t$

   




#558175 tuyển chọn các bài toán tính định thức

Đã gửi bởi sinh vien on 07-05-2015 - 12:35 trong Đại số tuyến tính, Hình học giải tích

Bài toán:(Putnam 2009 )Cho n > 3, n nguên dương. Tính định thức:

 $D_{n}=\begin{vmatrix} cos1 &cos2 &... &cosn \\ cos(n+1) & cos(n+2) &... &cos2n \\... &... &... &... \\cos(n^{2}-n+1) & cos(n^{2}-n+2) &... &cosn^{2} \end{vmatrix}$

Lời giải.  Lấy cột thứ 3 + cột thứ 1$\rightarrow$ cột thứ 1 và sử dụng biến đổi lượng giác , ta được :

$D_{n}=\begin{vmatrix} 2cos2cos1 &cos2 &... & ... &... cos(n) \\2cos(n+2)cos1 &cos(n+2) &... & ... &cos2n \\... & ... & & ... &... \\... &... & ... & ... &... \\2cos(n^{2}-n+2)cos1 & cos(n^{2}-n+2) &... &... & cosn^{2} \end{vmatrix}$

   Lúc này cột thứ 1= 2cos1 cột thứ 2 $\Rightarrow$ det$D_{n}=0$.

Bài toán . Xét các số phức$z_{1},z_{2},...,z_{2n}$ thỏa mãn điều kiện  $\left | z_{1} \right |=\left | z_{2} \right |=...=\left | z_{n+3} \right |$ và $argz_{1}\geqslant argz_{2}\geqslant ...\geq argz_{n+3}$.

  Tính định thức của ma trận B ,trong đó  $b_{ij}=\left | z_{i}-z_{j+n} \right |,i,j\in \left \{ 1,2,...n \right \}$.

 Lời giải . Ta có nhận xét sau bài toán đề cập đến modun và argument của một số phức nên ta sẽ sử dụng dạng lượng giác của số phức trong các tính toán. 

    Với hai số phức $z=r(cosx+isinx);\omega =r(cosy+isiny)\Rightarrow \left | z-\omega \right |=2r\left | sin\left ( \frac{x-y}{2} \right ) \right |$. 

  Kết hợp với điều kiện ở đầu bài, ta thấy B có dạng như sau:

$B=(2r)^{n}\begin{vmatrix} sin(x_{1}-x_{n+1}) &sin(x_{1}-x_{n+2}) & sin(x_{1}-x_{n+3}) & ...\\... &... & ... &... \\ sin(x_{n}-x_{n+1}) & sin\left ( x_{n}-x_{n+2} \right ) & sin(x_{n}-x_{n+3}) & ... \end{vmatrix}$

 trong đó $x_{i}$ kí hiệu tương ứng argument của $z_{i}$ và r=$\left | z_{1} \right |=\left | z_{2} \right |=...=\left | z_{2n} \right |$

 Nếu ta tách det B theo các cột thì dễ dàng nhận thấy det B=0 

 

 




#558174 tuyển chọn các bài toán tính định thức

Đã gửi bởi sinh vien on 07-05-2015 - 12:35 trong Đại số tuyến tính, Hình học giải tích

Bài toán:(Putnam 2009 )Cho n > 3, n nguên dương. Tính định thức:

 $D_{n}=\begin{vmatrix} cos1 &cos2 &... &cosn \\ cos(n+1) & cos(n+2) &... &cos2n \\... &... &... &... \\cos(n^{2}-n+1) & cos(n^{2}-n+2) &... &cosn^{2} \end{vmatrix}$

Lời giải.  Lấy cột thứ 3 + cột thứ 1$\rightarrow$ cột thứ 1 và sử dụng biến đổi lượng giác , ta được :

$D_{n}=\begin{vmatrix} 2cos2cos1 &cos2 &... & ... &... cos(n) \\2cos(n+2)cos1 &cos(n+2) &... & ... &cos2n \\... & ... & & ... &... \\... &... & ... & ... &... \\2cos(n^{2}-n+2)cos1 & cos(n^{2}-n+2) &... &... & cosn^{2} \end{vmatrix}$

   Lúc này cột thứ 1= 2cos1 cột thứ 2 $\Rightarrow$ det$D_{n}=0$.

Bài toán . Xét các số phức$z_{1},z_{2},...,z_{2n}$ thỏa mãn điều kiện  $\left | z_{1} \right |=\left | z_{2} \right |=...=\left | z_{n+3} \right |$ và $argz_{1}\geqslant argz_{2}\geqslant ...\geq argz_{n+3}$.

  Tính định thức của ma trận B ,trong đó  $b_{ij}=\left | z_{i}-z_{j+n} \right |,i,j\in \left \{ 1,2,...n \right \}$.

 Lời giải . Ta có nhận xét sau bài toán đề cập đến modun và argument của một số phức nên ta sẽ sử dụng dạng lượng giác của số phức trong các tính toán. 

    Với hai số phức $z=r(cosx+isinx);\omega =r(cosy+isiny)\Rightarrow \left | z-\omega \right |=2r\left | sin\left ( \frac{x-y}{2} \right ) \right |$. 

  Kết hợp với điều kiện ở đầu bài, ta thấy B có dạng như sau:

$B=(2r)^{n}\begin{vmatrix} sin(x_{1}-x_{n+1}) &sin(x_{1}-x_{n+2}) & sin(x_{1}-x_{n+3}) & ...\\... &... & ... &... \\ sin(x_{n}-x_{n+1}) & sin\left ( x_{n}-x_{n+2} \right ) & sin(x_{n}-x_{n+3}) & ... \end{vmatrix}$

 trong đó $x_{i}$ kí hiệu tương ứng argument của $z_{i}$ và r=$\left | z_{1} \right |=\left | z_{2} \right |=...=\left | z_{2n} \right |$

 Nếu ta tách det B theo các cột thì dễ dàng nhận thấy det B=0 

 

 




#558170 Hệ phương trình vi phân loại khó

Đã gửi bởi sinh vien on 07-05-2015 - 11:55 trong Giải tích

Bài toán ( Putnam 2009) . Gỉa sử $f,g,h$ là các hàm khả vi trên một khoảng mở chứa điểm 0 Và thỏa mãn :

$f^{'}=2f^{2}gh+\frac{1}{gh};g^{'}=fg^{2}h+\frac{4}{fh};h^{'}=3fgh^{2}+\frac{1}{fg}$ 

 và $f(0)=1;g(0)=1;h(0)=1$.

 Tìm một biểu thức cho hàm f(x) , giả thiết f(x) trong triệt tiêu trên khoảng mở chứa 0.

Lời giải. Nhân đẳng thức thứ nhất cho gh , thứ 2 cho fh, và đẳng thức cuối cho fg ta được

    $f^{'}gh=2\left ( fgh \right )^{2}+1;fg^{'}h=(fgh)^{2}+4;fgh^{'}=3(fgh)^{2}+1$.

 Cộng các đẳng thức này ta được

    $f^{'}gh+fg^{'}h+fgh^{'}=6(fgh)^{2}+6\Rightarrow (fgh)^{'}=6(fgh)^{2}+6$.

 Bằng cách đặt k(x)=f(x)g(x)h(x); ta được $k^{'}=6k^{2}+6\Rightarrow \frac{\mathrm{d} k}{\mathrm{d} x}=6k^{2}+6\Rightarrow \frac{dk}{6k^{2}+6}=dx$

 Kết hợp với điều kiện k(0)=1 ta thấy: $k(x)=tan\left ( 6x+\frac{\pi }{4} \right ).$

   Lại thấy:$\frac{f^{'}}{f}=2tan\left ( 6x+\frac{\pi }{4} \right )+cot\left ( 6x+\frac{\pi }{4} \right )$

$\Rightarrow lnf(x)=\frac{-2lncos\left ( 6x+\frac{\pi }{4} \right )+lnsin\left ( 6x+\frac{\pi }{4} \right )}{6}$

$\Rightarrow f(x)=e^{c}\left ( \frac{sin\left ( 6x+\frac{\pi }{4} \right )}{cos^{2}\left ( 6x+\frac{\pi }{4} \right )} \right )^{\frac{1}{6}}$. Từ giả thiết f(0)=1 ta tính được

   $f(x)=\frac{1}{2^{12}}\left ( \frac{sin\left ( 6x+\frac{\pi }{4} \right )}{cos^{2}\left ( 6x+\frac{\pi }{4} \right )} \right )^{\frac{1}{6}}$.

  Lưu ý là ta cũng có thể tính được g(x), h(x) bằng các tương tự.




#557941 Khai triển Fourier cho hàm Logaritm Gamma - Công thức Kummer

Đã gửi bởi sinh vien on 05-05-2015 - 15:39 trong Giải tích

Bài toán.Chứng minh rằng :

   $ln\Gamma (x)=\frac{1}{2}ln(2\pi )+\frac{1}{2}\sum_{k=1}^{\infty }\frac{cos2k\pi x}{k}+\sum_{k=1}^{\infty }\frac{(\gamma +ln2\pi k)sin2k\pi x }{\pi k}, (0< x< 1)$, trong đó $\gamma$ là hằng số Euler

Lời giải.  Để thực hiện được phép chứng minh trên ta cần đến các kết quả quen thuộc sau :

 $\Gamma (1-x)\Gamma (x)=\frac{\pi }{sin(\pi x)}$ ; $\int_{0}^{\pi }ln(sinx)dx=-\pi ln2$ và công thức

$\Gamma (x)=\frac{e^{-\gamma x}}{x}\prod_{k=1}^{\infty }e^{\frac{x}{k}}\left ( 1+\frac{x}{k} \right )^{-1}$

  Ta có:

 $a_{0}=2\int_{0}^{1}ln\Gamma (x)dx=2\int_{1}^{0}ln\Gamma (1-x)d(1-x)=2\int_{0}^{1}ln\Gamma (1-x)dx$

$\Rightarrow a_{0}=\int_{0}^{1}ln\left ( \frac{\pi }{sin\pi x} \right )dx=ln\pi -\int_{0}^{1}ln(sin\pi x)dx=ln(2\pi )$

$a_{k}=2\int_{0}^{1}ln\Gamma (x)cos2k\pi xdx=2\int_{0}^{1}ln\Gamma (1-x)cos2k\pi xdx$

$\Rightarrow a_{k}=\int_{0}^{1}ln\left ( \frac{\pi }{sin\pi x} \right )cos2k\pi xdx=-\int_{0}^{1}ln(sin\pi x)cos2k\pi xdx$$=-\frac{1}{\pi }\int_{0}^{\pi }ln(sinx)cos2kxdx=-ln(sinx)\frac{sin2k x}{2k\pi } _{0}^{\pi } +\frac{1}{2k\pi }\int_{0}^{\pi }\frac{sin2k xcosx}{sinx}dx$=$\frac{1}{2k\pi }\int_{0}^{\pi }\frac{sin2kxcosxdx}{sinx}$

Đặt $I_{k}=\int_{0}^{\pi }\frac{sin2kxcosxdx}{sinx}\Rightarrow I_{k+1}=\int_{0}^{\pi }\frac{sin2(k+1)xcosxdx}{sinx}$. Do đó $I_{k+1}-I_{k}=2\int_{0}^{\pi }\frac{cos(2k+1)xsinxcosxdx}{sinx}=2\int_{0}^{\pi }cos(2k+1)xcosxdx=0$ nên $I_{k}=2\int_{0}^{\pi }cos^{2}xdx=\pi \Rightarrow a_{k}=\frac{1}{2k\pi }\times \pi =\frac{1}{2k}$

  Theo một kết quả quen thuộc:

                              $\Gamma (x)=\frac{e^{-\gamma x}}{x}\prod_{k=1}^{\infty }e^{\frac{x}{k}}\left ( 1+\frac{x}{k} \right )^{-1}\Rightarrow ln\Gamma (x)=-(\gamma x+lnx)+\sum_{k=1}^{\infty }\left \{ \frac{x}{k}-ln\left ( 1+\frac{x}{k} \right ) \right \}$$\Rightarrow ln\Gamma (x)sin2k\pi x=-(\gamma x+lnx)sin2k\pi x+\sum_{k=1}^{\infty }\left \{ \frac{x}{k}-ln\left ( 1+\frac{x}{k} \right ) \right \}sin2k\pi x$.

  trong đó :

       $\int_{0}^{1}(-\gamma x-lnx)sin2k\pi xdx=\frac{\gamma xcos2k\pi x}{2k\pi }_{0}^{1}-\frac{\gamma }{2k\pi }\int_{0}^{1}cos2k\pi xdx+\frac{lnxcos2k\pi x}{2k\pi }_{0}^{1}-\frac{1}{2k\pi }\int_{0}^{1}\frac{cos2k\pi xdx}{x}=\frac{\gamma }{2k\pi }+\left [ \frac{lnxcos2k\pi x-Ci(2k\pi x)}{2k\pi } \right ]_{0}^{1}$$\frac{\gamma -Ci(2k\pi )}{2k\pi }-lim_{\epsilon \rightarrow \infty }\frac{ln\epsilon -Ci(2k\pi \epsilon )}{2k\pi }=\frac{\gamma -Ci(2k\pi )}{2k\pi }-lim_{\epsilon \rightarrow \infty }\frac{ln\epsilon -ln(2k\pi )-ln\epsilon -\gamma +O(1)}{2k\pi }$$=\frac{2\gamma +ln(2k\pi )-Ci(2k\pi )}{2k\pi }$

      $\int_{0}^{1}\sum_{k=1}^{\infty }\left \{ \frac{x}{k}-ln(1+\frac{x}{k}) \right \}sin2k\pi xdx=-\frac{1}{2k\pi }\sum_{n=1}^{\infty }\left \{ \frac{1}{n}+Ci(2k\pi n+2k\pi )-Ci(2k\pi n)-ln\left ( 1+\frac{1}{n} \right ) \right \}$

$=-\frac{1}{2k\pi }\left [ \sum_{n=1}^{\infty } \left \{ \frac{1}{n}-ln\left ( 1+\frac{1}{n} \right ) \right \}+lim_{N\rightarrow \infty }\left ( \sum_{n=2}^{N+1}Ci(2k\pi n)-\sum_{n=1}^{N}Ci(2k\pi n) \right )\right ]$

$=-\frac{1}{2k\pi }\left [ \gamma +lim_{N\rightarrow \infty } \left ( Ci(2k\pi x+2k\pi N) -Ci(2k\pi )\right )\right ]=\frac{Ci(2k\pi )-\gamma }{2k\pi }$.

  Do đó

                $\int_{0}^{1}ln\Gamma (x)sin2k\pi xdx=\frac{\gamma +ln(2k\pi )}{2k\pi }\Rightarrow b_{k}=\frac{\gamma +ln(2k\pi )}{k\pi }$




#557905 Ứng dụng giải tích trong các bài toán số học

Đã gửi bởi sinh vien on 05-05-2015 - 08:46 trong Giải tích

Bài toán.  Cho n là một số nguyên lớn hơn bằng 2.

Chứng minh rằng

 $\sum_{p\leq n}\frac{1}{p}> lnlnn-1$, trong đó ta quy ước p chỉ các số nguyên tố nên tổng ở đây lấy trên tập các số nguyên tố nhỏ hơn bằng n.

Lời giải. Gọi A(n) là tập các số nguyên dương sao cho các ước số nguyên tố nhỏ hơn hoặc bằng n.

Khi đó

 $\prod_{p\leq n}\left ( 1+\frac{1}{p}+\frac{1}{p^{2}}+...+\frac{1}{p^{k}}+.... \right )=\sum_{m\in A(n)}\frac{1}{m}$ (1)

Ta thấy rằng $\sum_{m\in A(n)}\frac{1}{m}\geq \sum_{m=1}^{n}\frac{1}{m}$ (định nghĩa của tập A(n) )

mà ta lại có bất đẳng thức sau ( chứng minh không quá khó khăn )   $\sum_{m=1}^{n}\frac{1}{m}> lnn$  . Lưu ý rằng ta cũng có đẳng thức sau :$1+\frac{1}{p}+\frac{1}{p^{2}}+...+\frac{1}{p^{k}}+...=\left (1-\frac{1}{p} \right )^{-1}$ nên ta có thể viết lại  (1) như sau$\prod_{p\leq n}\left ( 1-\frac{1}{p} \right )^{-1}> lnn$.

 Để thực hiện tiếp phép chứng minh ta cần đến một bất đẳng thức không tầm thường :

$e^{t+t^{2}}\geq \left ( 1-t \right )^{-1}, 0\leq t\leq \frac{1}{2}$ có thể được chứng minh dễ dàng bằng  phương pháp lấy đạo hàm :

  Do đó :

    $\prod_{p\leq n}e^{\frac{1}{p}+\frac{1}{p^{2}}}\geq \prod_{p\leq n}\left ( 1-\frac{1}{p} \right )^{-1}> lnn$.(2)

 Từ (2) không quá khó để nhận ra rằng $\sum_{p\leq n}\frac{1}{p}+\sum_{p\leq n}\frac{1}{p^{2}}>lnlnn\Rightarrow \sum_{p\leq n}\frac{1}{p}> lnlnn-\sum_{p\leq n}\frac{1}{p^{2}}$

 mà $\sum_{p\leq n}\frac{1}{p^{2}}\leq \sum_{k=2}^{\infty }\frac{1}{k^{2}}=\frac{\pi ^{2}}{6}-1< 1$ (3)

  Từ (2) và (3) ta đi đến :$\sum_{p\leq n}\frac{1}{p}> lnlnn-1$ (đpcm)

         




#557834 CMR tồn tại hằng số B dương thỏa $\sum_{i,j=1}^{n...

Đã gửi bởi sinh vien on 04-05-2015 - 18:56 trong Giải tích

Bài toán. (PUTNAM 2011) Cho $a_{1},a_{2},...a_{n}$ là các số thực thỏa mãn 

          $\int_{-\infty }^{\infty }\left ( \sum_{i=1}^{n}\frac{1}{1+\left ( x-a_{i} \right )^{2}}\right )^{2}dx\leqslant An$ với mọi n , trong đó A là một hằng số dương .

 Chứng minh rằng tồn tại hằng số B dương thỏa $\sum_{i,j=1}^{n}\left ( 1+\left ( a_{i}-a_{j} \right )^{2} \right )\geq Bn^{3}$.

Lời giải. 

 Với $a\neq 0$, $\int_{-\infty }^{\infty }\frac{du}{\left ( 1+\left ( u+a \right )^{2} \right )\left ( 1+\left ( u-a \right )^{2} \right )}=\frac{1}{4a(1+a^{2})}\int_{-\infty }^{\infty }\left [ \frac{u+a}{1+\left ( u+a\right )^{2}}-\frac{u-a}{1+\left ( u-a \right )^{2}} \right ]du$$=\frac{1}{4a\left ( 1+a^{2} \right )}\left [ \frac{1}{2}ln\frac{1+\left ( u+a \right )^{2}}{1+(u-a)^{2}} +aarctan(u+a)+aarctan(u-a)\right ]_{-\infty }^{\infty }$$=\frac{\pi }{2(1+a^{2})}$.

   Sử dụng phép đổi biến: $x=u+\frac{a+b}{2}$ ta được 

$\int_{-\infty }^{\infty }\frac{dx}{(1+(x-a)^{2})(1+(x-b)^{2})}=\frac{2\pi }{4+(a-b)^{2}}$, trong đó $a\neq b$

Trong trường hợp a=b 

  $\int_{-\infty }^{\infty }\frac{dx}{(1+(x-a)^{2})^{2}}=\int_{-\frac{\pi }{2}}^{\frac{\pi }{2}}\frac{\frac{1}{cos^{2}\theta }d\theta }{\frac{1}{cos^{4}\theta }}=\int_{-\frac{\pi }{2}}^{\frac{\pi }{2}}cos^{2}\theta d\theta =\frac{\pi }{2}$

 nên côn thức trên cũng đúng với mọi cặp số a,b.

 Ta có

$An\geq \int_{-\infty }^{\infty }\left [ \sum_{i=1}^{n}\frac{1}{1+(x-a_{i})^{2}}\right ]^{2}dx=\sum_{i,j=1}^{n}\int_{-\infty }^{\infty }\frac{dx}{(1+(x-a_{i})^{2})(1+(x-a_{j})^{2})}$$=\sum_{i,j=1}^{n}\frac{2\pi }{4+(a_{i}-a_{j})^{2}}\geq \frac{\pi }{2}\sum_{i,j=1}^{n}\frac{1}{1+\left ( a_{i}-a_{j} \right )^{2}}$

 Sử dụng bất đẳng thức quen thuộc Cauchy- Schwarz 

$n^{4}\leq \left [ \sum_{i,j=1}^{n}\frac{1}{1+(a_{i}-a_{j})^{2}} \right ]\sum_{i,j=1}^{n}(1+(a_{i}-a_{j})^{2})\leq \frac{2An}{\pi }\sum_{i,j=1}^{n}(1+(a_{i}-a_{j})^{2})$

  Chọn $B=\frac{\pi }{2A}$ . thì điều kiện bài toán được thỏa mãn




#557831 Một sự kết hợp giữa đại số tuyến tính và tổ hợp

Đã gửi bởi sinh vien on 04-05-2015 - 18:29 trong Đại số tuyến tính, Hình học giải tích

Bài toán.  Cho $k\leq \frac{n}{2}$ và $F$ là một họ các ma trận con của một ma trận $n\times n$  sao cho hai ma trận con bất kì đều giao nhau ( có những phần tử chung ) thì $\left | F \right |\leq \left ( C_{n-1}^{k-1} \right )^{2}$, trong đó $\left | A \right |$ kí hiệu số phần tử của tập A.

Lời giải.

Với mọi ma trận con M thuộc họ F, đặt $R_{M};C_{M}$ là các bộ k- số  chỉ thứ tự của các hàng và các cột . Dể dàng nhận thấy $R_{M};C_{M}$ xác định duy nhất ma trận M. Theo giả thiết của bài toán

      $R_{M_{1}}\bigcap R_{M_{2}}\neq \varnothing ;C_{M_{1}}\bigcap C_{M_{2}}\neq \varnothing$ trong đó $M_{1};M_{2}$ là hai phần tử bất kì thuộc họ F.

 Khi đó nếu đặt   $R=\left \{ R_{M};M\in F \right \}$;$C=\left \{ C_{M};M\in F \right \}$ là hai  họ con chứa các bộ k -số lấy từ n số sao cho hai bộ k- số bất kỳ đều có phần tử chung .

   Theo định lý Erdos-Ko-Rado ( xem tuyển tập các chuyên đề tổ hợp ) ta thấy

                 $\left | R \right |\leq C_{n-1}^{k-1};\left | C \right |\leq C_{n-1}^{k-1}\textrm{}$

Theo nhận xét trên ta suy ra$\left | F \right |\leq \left ( C_{n-1}^{k-1}\right )^{2}$ (đpcm )




#557829 Tính tích phân hai lớp bằng tổng Rieman $I=\int \int_{D...

Đã gửi bởi sinh vien on 04-05-2015 - 17:59 trong Giải tích

Bài toá n. Tính tích phân

   

    $I=\int \int_{D} ln\left | sin(x-y) \right |dxdy$, trong đó $D=\left \{ (x,y)/0\leq x< y\leqslant \pi \right \}$

Lời giải:

    Để tính tích phân hai lớp này, trước hết ta chứng minh đẳng thức sau:

   $\sum_{0\leqslant j< k\leq n-1}\frac{\pi ^{2}}{n^{2}}ln\left | sin\left ( \frac{j\pi }{n}-\frac{k\pi }{n} \right ) \right |=\frac{\pi ^{2}}{2n}lnn-\left ( 1-\frac{1}{n} \right )\frac{\pi ^{2}}{2}ln2$.

  Xét định thức:

     $A=\begin{vmatrix} 1 & 1 & 1 & ... & 1\\ 1& \varepsilon &\varepsilon ^{2} &...& \varepsilon ^{n-1}\\ 1& \varepsilon ^{2} &\varepsilon ^{4} &... &\varepsilon ^{2(n-1)} \\ ... & ... & .... & ... & ...\\ 1 & \varepsilon ^{n-1} &\varepsilon ^{2(n-1)} &... &\varepsilon ^{(n-1)(n-1)} \end{vmatrix}$

   trong đó $\varepsilon =e^{\frac{2\pi i}{n}}$

 Ta thấy đây là định thức Vandermonde nên 

  $A=\prod_{0\leq j< k\leq n-1}\left ( \varepsilon ^{j}-\varepsilon ^{k} \right )$.

 Mặt khác  liên hợp :$\bar{A}=\begin{vmatrix} 1 & 1 & 1 & ... & 1\\ 1& \bar{\varepsilon } &\bar{\varepsilon }^{2} &... &\bar{\varepsilon}^{n-1} \\1 & \bar{\varepsilon }^{2} & \bar{\varepsilon } ^{4}& ... & \bar{\varepsilon }^{2(n-1)}\\ ... & ... & ...& ... & \\ 1& \bar{\varepsilon }^{n-1} &\bar{\varepsilon }^{2(n-1)} &... & \bar{\varepsilon }^{(n-1)(n-1)} \end{vmatrix}$

 Nên

          $A\bar{A}=\begin{vmatrix} n& 0 & 0 &... &0 \\ 0 & n & 0 & ...& 0\\... & ...& ...& ...& ...\\ ... & ... & ... & ... &... \\ 0& 0& 0 & ... &n \end{vmatrix}=n^{n}$

$\Rightarrow \prod_{0\leq j< k\leq n-1}\left ( \varepsilon ^{j}-\varepsilon ^{k} \right )\prod_{0\leq j< k\leq n-1}\overline{(\varepsilon ^{j}-\varepsilon ^{k})}=n^{n}$

$\prod_{0\leq j< k\leq n-1}\left | \varepsilon ^{j}-\varepsilon ^{k}\right |^{2}=n^{n}$.

 Ta thấy:

     $\left |\varepsilon ^{j} -\varepsilon ^{k} \right |^{2}=\left ( cos\frac{2\pi j}{n} -cos\frac{2\pi k}{n}\right)^{2}+\left (sin\frac{2\pi j}{n} -sin\frac{2\pi k}{n}\right )^{2}$

        $=2\left ( 1-cos\frac{2\pi (j-k)}{n} \right )=\left [ 2sin\left ( \frac{j\pi }{n}-\frac{k\pi }{n} \right ) \right ]^{2}$

    hay $\prod_{0\leq j< k\leq n-1}\left [ 2sin\left ( \frac{j\pi }{n}-\frac{k\pi }{n} \right )\right ]^{2}=n^{n}$

  Lấy logarithm hai vế và sử dụng một chút biến đổi ta được đẳng thức đã nêu.

   Dễ dàng nhận thấy rằng vế trái của đẳng thức này là tổng Riemann của tích phân cần tính ở đây ta đã sử dụng  phân hoạch đều:

     Do đó:

  $\int \int_{D}ln\left | sin(x-y) \right |dxdy=lim_{n\rightarrow \infty }\left [ \frac{\pi ^{2}}{2n} lnn-\left ( 1-\frac{1}{n} \right )\frac{{\pi ^{2}}}{2}ln2\right ]$$=-\frac{\pi ^{2}}{2}ln2$.

  




#557803 Tính tích phân: $\iiint {x^2+y^2+z^2} dxdydz $, V là...

Đã gửi bởi sinh vien on 04-05-2015 - 09:55 trong Giải tích

Đặt $x=\frac{1}{2}+rsin\varphi cos\theta ;y=\frac{1}{2}+rsin\varphi sin\theta ;z=\frac{1}{2}+rcos\varphi$;

trong đó $0\leq r\leq \frac{\sqrt{3}}{2};0\leq \theta \leq 2\pi ;0\leq \varphi \leq \pi$. Định thức hàm $J=r^{2}sin\varphi$

 Do đó $\int_{0}^{2\pi }d\theta \int_{0}^{\pi }d\varphi \int_{0}^{\frac{\sqrt{3}}{2}}(\frac{3}{2}+rsin\varphi cos\theta +rsin\varphi sin\theta +rcos\varphi )r^{2}sin\varphi dr$. Tách tích phân trên thành 4 phần:

$I_{1}=\frac{3}{2}\int_{0}^{2\pi }d\theta \int_{0}^{\pi }sin\varphi d\varphi \int_{0}^{\frac{\sqrt{3}}{2}}r^{2}dr$ 

$I_{2}=\int_{0}^{2\pi }cos\theta d\theta \int_{0}^{\pi }sin^{2}\varphi d\varphi \int_{0}^{\frac{\sqrt{3}}{2}}r^{3}dr$

$I_{3}=\int_{0}^{2\pi }sin\theta d\theta \int_{0}^{\pi }sin^{2}\varphi d\varphi \int_{0}^{\frac{\sqrt{3}}{2}}r^{3}dr$

$I_{4}=\int_{0}^{2\pi }d\theta \int_{0}^{\pi }cos\varphi sin\varphi d\varphi \int_{0}^{\frac{\sqrt{3}}{2}}r^{3}dr$.

 Ta thấy $\int_{0}^{2\pi }cos\theta d\theta =\int_{0}^{2\pi }sin\theta d\theta =\int_{0}^{\pi }cos\varphi sin\varphi d\varphi =0$ nên

 

$I=\frac{3}{2}\times 2\pi \times 2\times \frac{\sqrt{3}}{8}$